Inscription / Connexion Nouveau Sujet
Niveau Maths sup
Partager :

arithmétique

Posté par
Deb
18-11-17 à 14:21

Bonjour à tous,
j'ai des exercices à préparer qui n'ont rien à voir avec ce que l'on fait en cours mais le prof a dit que ça permettrait de se rappeler de certaines méthodes de terminale et que l'on pouvait chercher de l'aide sur internet. Bref voici la 1ère question:
trouver le dernier chiffre de (19871991)1993.
Alors je me suis dis qu'on pouvait peut-être commencer avec 1987 congru à 1 modulo 10. et sur internet certains parlaient d'utiliser Euler...
C'est un peu (très) flou et j'aurai aimé que quelqu'un m'explique la méthode. Merci !

Posté par
ThierryPoma
re : arithmétique 18-11-17 à 14:49

Bonjour,

Citation :
commencer avec 1987 congru à 1 modulo 10


Ah bon ? Tu en es certain ?

Posté par
Deb
re : arithmétique 18-11-17 à 14:50

non pas du tout :') j'avoue avoir oublié tout ça

Posté par
ThierryPoma
re : arithmétique 18-11-17 à 14:52

Si j'écris que 1987=198\times10+7, avec 0\leqslant7<10, cela te fait-il penser à quelque chose ?

Posté par
Deb
re : arithmétique 18-11-17 à 14:55

eh bien que 1987 congru à 7 modulo 10

Posté par
ThierryPoma
re : arithmétique 18-11-17 à 15:03

Donc,

1987^{1991}\equiv7^{1991}=7^{1990}\times7=\left(7^2\right)^{995}\times7\equiv\cdots\quad[10]

Posté par
ThierryPoma
re : arithmétique 18-11-17 à 15:14

Alors ?

Posté par
Deb
re : arithmétique 18-11-17 à 15:15

je ne vois pas trop quelle autre étape faire après...

Posté par
ThierryPoma
re : arithmétique 18-11-17 à 15:25

1987^{1991}\equiv7^{1991}=7^{1990}\times7=\left(7^2\right)^{995}\times7\equiv\left(49\right)^{995}\times7\equiv\left(50-1\right)^{995}\times7\equiv\left(-1\right)^{995}\times7\equiv-7\equiv-7+10\equiv3\quad[10]

Partant,

\left(1987^{1991}\right)^{1993}\equiv3^{1993}=\cdots\quad[10]

Posté par
Deb
re : arithmétique 18-11-17 à 15:44

Ah merci beaucoup ! alors je vais faire la 2nde partie et vous l'envoyer pour que vous me disiez si ça va. Mais d'abord je ne comprend pas la  dernière étape du calcul: pourquoi écrivez vous "-7+10" ?

Posté par
ThierryPoma
re : arithmétique 18-11-17 à 15:46

Penses-tu que -7 puisse être le chiffre des unités de 1987^{1991} ?

Posté par
ThierryPoma
re : arithmétique 18-11-17 à 15:57

Erratum :

Désolé, mais il faut lire ceci :

(\cdots)\equiv-7\equiv-7+10=3\quad[10]

Posté par
Deb
re : arithmétique 18-11-17 à 16:29

Alors voilà ce que j'ai fait pour la suite:
(1987^{991})^{1993}\equiv 3^{1993} [10] = 3^{1992} \times 3 = (3^{2})^{996} \times 3 = (10 - 1)^{996} \times 3 = (-1)^{996} \times 3 = 3 [10]
c'est bien ça ?

Posté par
ThierryPoma
re : arithmétique 18-11-17 à 16:37

Rectif :

(1987^{1991})^{1993}\equiv3^{1993}=3^{1992}\times3=(3^{2})^{996}\times3=(10 - 1)^{996}\times3\equiv(-1)^{996}\times3=3\quad[10]

Posté par
Deb
re : arithmétique 18-11-17 à 16:42

ok merci beaucoup ! et du coup le dernier chiffre est un 3 ?

Posté par
ThierryPoma
re : arithmétique 18-11-17 à 16:50

Sauf erreur, oui. Que veux-tu que ce soit d'autre ? Il me semble que 0\leqslant3\leqslant9, non ?

Dit autrement, 3 est le reste dans la division euclidienne de (1987^{1991})^{1993} par 10...

Posté par
Deb
re : arithmétique 18-11-17 à 16:55

ok merci beaucoup pour le temps que vous avez pris pour moi !
Est-ce que je peux vous poser une autre question ou il faut forcément que j'ouvre un nouveau post ?

Posté par
ThierryPoma
re : arithmétique 18-11-17 à 16:57

S'il s'agit d'une question qui porte sur le même problème, oui. Sinon, il faut ouvrir un autre fil, conformément aux règles du forum.

Posté par
Deb
re : arithmétique 18-11-17 à 16:59

C'est dans le même exercice mais ce n'est pas lié à cette question. Bon je vais en ouvrir un autre je pense que ce sera mieux.
Merci beaucoup encore une fois !

Posté par
ThierryPoma
re : arithmétique 18-11-17 à 17:05

Si c'est dans le même exo, je te conseille vivement de poursuivre ici. A moins que je ne me trompe...

Posté par
Deb
re : arithmétique 18-11-17 à 17:17

ok bon je vais tenter ma chance alors . La question suivante est "trouver tous les couples (a,b) d'entiers strictement positifs tels que ba=ab"
J'ai tout d'abord démontré que ba=ab ssi \frac{ln(a)}{a} = \frac{ln(b)}{b} mais après je suis bloquée.

Posté par
ThierryPoma
re : arithmétique 18-11-17 à 17:43

L'on a clairement (a,\,b)\in\{(1,\,1),\,(2,\,2),\,(2,\,4),\,(4,\,2)\}. Sont-ce les seules ? Une étude de la fonction x\mapsto\dfrac{\ln\,x}{x} sur \R^{*+} pourrait t'aider...

Posté par
flight
re : arithmétique 18-11-17 à 19:04

salut

on pouvait partir de  74 = 1[10]   alors  (74 )497= 1[10]

alors  (74 )497.73= 73[10]   soit  

(74 )497.73= 3[10]    alors

71991 = 3[10]     , 719911993 = 31993[10]

on sait que  34=1[10]   et que  1993=4*498+1  alors

31993 = 3[10]...  alors  719911993= 3[10)

Posté par
carpediem
re : arithmétique 19-11-17 à 10:02

salut

Deb @ 18-11-2017 à 17:17

ok bon je vais tenter ma chance alors . La question suivante est "trouver tous les couples (a,b) d'entiers strictement positifs tels que ba=ab"
J'ai tout d'abord démontré que ba=ab ssi \frac{ln(a)}{a} = \frac{ln(b)}{b} mais après je suis bloquée.

non tu n'as rien démontré ...

tu appliques simplement la propriété triviale : pour toute fonction f : si a = b alors f(a) = f(b) qui est une trivialité par définition d'une fonction (tout réel possède au plus une image)

par contre tout le pb c'est la réciproque : si f(a) = f(b) a-t-on (uniquement que) a = b ?

on voit donc l'intéret d'introduire une fonction ici pour résoudre le pb demandé ...

Posté par
Deb
re : arithmétique 19-11-17 à 13:07

mais je nevois pas vraiment porquoi fait il faire une étude de fonction... Enfin bon je l'ai fait et j'ai trouvé ça: (je ne suis pas sure de moi)
\begin{array} {|c|cccccc|} x &0 & & e & & +\infty & \\ {signe} & & + & 0 & - & & \\ {variation} & & \nearrow & & \searrow & & \end{array}

Posté par
Deb
re : arithmétique 19-11-17 à 13:07

(avec 0 une valeur interdite)

Posté par
carpediem
re : arithmétique 19-11-17 à 13:17

ce tableau ne te montre-t-il pas que si 0 < y < e alors l'équation f() = y admet deux antécédents

maintenant si on cherche des solutions entières avec y entier ben yapa 36000 issues ...

Posté par
Deb
re : arithmétique 19-11-17 à 13:26

non je suis désolée mais je suis vraiment à côté de la plaque... Déjà que voulez-vous dire par y ?

Posté par
carpediem
re : arithmétique 19-11-17 à 13:31

carpediem @ 19-11-2017 à 13:17

ce tableau ne te montre-t-il pas que si 0 < y < f(e) alors l'équation f() = y admet deux antécédents

maintenant si on cherche des solutions entières avec y entier ben yapa 36000 issues ...


ne vois-tu pas ce qu'est y ? ...

Posté par
Deb
re : arithmétique 19-11-17 à 13:44

il s'agit de ln x/x ?

Posté par
Deb
re : arithmétique 19-11-17 à 15:35

donc tous les entiers compris entre 0 et e permettent l'égalité ba=ab ?

Posté par
carpediem
re : arithmétique 19-11-17 à 16:00

Posté par
Deb
re : arithmétique 19-11-17 à 16:38

bon j'abandonne... Merci pour votre aide

Posté par
carpediem
re : arithmétique 19-11-17 à 17:00

de rien



les seuls entiers entre 1 et e sont 1, 2, et 3 et ce sont donc les candidats

quelles sont les solutions des équations :

a/  1^b = b^1
 \\ b/  2^b = b^2
 \\ c/  3^b = b^3

Posté par
ThierryPoma
re : arithmétique 19-11-17 à 18:20

Bonsoir tout le monde,

@Carpi : Bonsoir ! Es-tu certain que 3<e ?

Posté par
carpediem
re : arithmétique 19-11-17 à 18:48

Posté par
Deb
re : arithmétique 19-11-17 à 20:15

mais sauf que le couple (2,4) marche mais 4 est supérieur à e...

Posté par
ThierryPoma
re : arithmétique 19-11-17 à 20:24

Il est clair que

S=\{(2,\,4),\,(4,\,2)\}\cup\bigcup_{n\in\N^*}\{(n,\,n)\}

est l'ensemble des solutions entières de a^b=b^a. En effet, le sous-ensemble \{(2,\,4),\,(4,\,2)\} de S résulte de l'étude de la fonction x\mapsto\dfrac{\ln\,x}{x} sur \R^{*+} et de ce que

\dfrac{\ln\,2}{2}=\dfrac{2\times\ln\,2}{2\times2}=\dfrac{\ln\,4}{4}\mbox{, avec }2<e<4

Posté par
Deb
re : arithmétique 19-11-17 à 20:34

mais alors comment trouver les couples ?

Posté par
ThierryPoma
re : arithmétique 19-11-17 à 20:48

Mis à part les solutions triviales données par

\bigcup_{n\in\N^*}\{(n,\,n)\}

existe-t-il un couple d'entiers naturels (u,\,v) au moins tels que

\dfrac{\ln\,u}{u}=\dfrac{\ln\,v}{v}\mbox{, avec }u<e<v\mbox{ et }u\ne2\mbox{ et }v\ne4

Posté par
Deb
re : arithmétique 19-11-17 à 22:49

eh bien non je ne pense pas

Posté par
ThierryPoma
re : arithmétique 20-11-17 à 08:20

Bonjour,

Ce n'est pas ce que tu penses qui est important ici, mais c'est de prouver le résultat voulu en s'appuyant sur les variations de la fonction x\mapsto\dfrac{\ln\,x}{x} sur \R^{*+} et de ce que dans l'intervalle ]1,\,e[ il n'existe pas beaucoup d'entiers naturels.

Fais une figure pour comprendre...

Posté par
carpediem
re : arithmétique 20-11-17 à 19:01

c'est ce que je disais à 17h00 sous l'aval de ta correction bien sur ...



Vous devez être membre accéder à ce service...

Pas encore inscrit ?

1 compte par personne, multi-compte interdit !

Ou identifiez-vous :


Rester sur la page

Inscription gratuite

Fiches en rapport

parmi 1675 fiches de maths

Désolé, votre version d'Internet Explorer est plus que périmée ! Merci de le mettre à jour ou de télécharger Firefox ou Google Chrome pour utiliser le site. Votre ordinateur vous remerciera !